Difference between revisions of "1987 AJHSME Problems/Problem 3"

m (Solution)
Line 8: Line 8:
  
 
<math>2(81+83+85+87+89+91+93+95+97+99)</math>
 
<math>2(81+83+85+87+89+91+93+95+97+99)</math>
 
+
The answer is E
 
<math>\begin{align*}
 
<math>\begin{align*}
 
& = 2\Big( (81+99) + (83+97) + (85+95) + (87+93) + (89+91) \Big) \\
 
& = 2\Big( (81+99) + (83+97) + (85+95) + (87+93) + (89+91) \Big) \\

Revision as of 20:07, 26 October 2015

Problem

$2(81+83+85+87+89+91+93+95+97+99)=$

$\text{(A)}\ 1600 \qquad \text{(B)}\ 1650 \qquad \text{(C)}\ 1700 \qquad \text{(D)}\ 1750 \qquad \text{(E)}\ 1800$

Solution

$2(81+83+85+87+89+91+93+95+97+99)$ The answer is E $\begin{align*} & = 2\Big( (81+99) + (83+97) + (85+95) + (87+93) + (89+91) \Big) \\ & = 2(180+180+180+180+180) \\ & = 2\cdot 5\cdot 180 \\ & = 1800\rightarrow \boxed{\text{E}} \end{align*}$ (Error compiling LaTeX. Unknown error_msg)

See Also

1987 AJHSME (ProblemsAnswer KeyResources)
Preceded by
Problem 2
Followed by
Problem 4
1 2 3 4 5 6 7 8 9 10 11 12 13 14 15 16 17 18 19 20 21 22 23 24 25
All AJHSME/AMC 8 Problems and Solutions

The problems on this page are copyrighted by the Mathematical Association of America's American Mathematics Competitions. AMC logo.png